ONEM 2018 - Fase 3 - Nivel 3 - P8

Problemas que aparecen en el Archivo de Enunciados.
luisq
Mensajes: 20
Registrado: Jue 25 Oct, 2018 10:45 am
Nivel: 3

ONEM 2018 - Fase 3 - Nivel 3 - P8

Mensaje sin leer por luisq »

En un torneo de vóley participaron $k$ equipos y cada equipo se enfrentó a cada uno de los otros exactamente una vez. Al final del torneo se notó que exactamente el $80\%$ de los equipos ganó al menos $2$ partidos. Calcule la suma de todos los posibles valores de $k$.

Aclaración: Considere que en el vóley un partido no puede quedar en empate.
Fedex

COFFEE - Mención-COFFEE Matías Saucedo OFO - Medalla de Plata-OFO 2020 FOFO Pascua 2020 - Medalla-FOFO Pascua 2020 COFFEE - Mención-COFFEE Ariel Zylber COFFEE - Mención-COFFEE Iván Sadofschi
FOFO 10 años - Medalla-FOFO 10 años OFO - Medalla de Plata-OFO 2021 OFO - Jurado-OFO 2022 OFO - Jurado-OFO 2023 FOFO 13 años - Jurado-FOFO 13 años
OFO - Jurado-OFO 2024
Mensajes: 269
Registrado: Mar 31 Dic, 2019 2:26 am
Medallas: 11
Nivel: 3
Ubicación: Rosario, Santa Fe
Contactar:

Re: ONEM 2018 - Fase 3 - Nivel 3 - P8

Mensaje sin leer por Fedex »

Puse personas en vez de equipos xd
Spoiler: mostrar
El problema nos dice que $\frac{4}{5}k$ es un entero. Luego $k = 5n$ para algún entero $n$.
Este $n$ representa la cantidad de personas que ganaron $0$ o $1$ partido.

Podemos ver que dentro del torneo no pueden haber $2$ personas (o más) que hayan perdido todos sus partidos, ya que de esta forma ambos deberían haber perdido el partido que jugaron entre ellos, absurdo.

Ahora veamos que no pueden haber $4$ personas (o más) que hayan ganado $1$ partido.
Llamemos a estos $A$, $B$, $C$ y $D$.
Cómo $A$ solo ganó un partido, dentro del grupo $B$, $C$ y $D$ existen al menos $2$ a los que $A$ no les ganó, o sea que le ganaron a $A$, supongamos $B$ y $C$. Ambos ganaron su único partido contra $A$ por lo que entonces $B$ perdió su partido con $C$ y viceversa, absurdo.

Luego $n \leq 1 + 3 = 4$.

Si $n = 4$ entonces necesariamente hay $1$ persona $A$ que perdió todos sus partidos y $3$ que ganaron solo uno (el partido contra $A$). Luego por un argumento similar las personas del grupo de $3$ perdieron todos los partidos que jugaron entre ellos, absurdo.

Ahora $n = 3$ es posible con $3$ que hayan ganado un solo partido $A$, $B$ y $C$ de forma que $A$ le ganó a $B$, $B$ le ganó a $C$ y $C$ le ganó a $A$, perdiendo el resto de partidos con las otras $12$ personas, que ganaron a lo sumo $3$ partidos cada uno.

Para $n = 2$ es posible si tomamos una persona que haya perdido todos sus partidos y una que haya ganado uno solo (al que los perdió todos). Luego las otras $8$ personas ganaron al menos $2$ partidos contra los anteriores.

Finalmente $n = 1$ es posible si tomamos una persona que los perdiera todos, dandole un partido ganado a los otros $4$. Y en este grupo hacemos un cuadrado en el que $A$ le ganó a $B$, $B$ a $C$, $C$ a $D$ y $D$ a $A$ dándoles a cada uno una victoria más.

Luego el valor pedido es: $5 + 10 + 15 = 30$.
This homie really did 1 at P6 and dipped.
Responder